王进明 初等数论 习题解答 下载本文

王进明 初等数论 习题及作业解答

P17 习题1-1 1,2(2)(3), 3,7,11,12为作业。

1.已知两整数相除,得商12,余数26,又知被除数、除数、商及余数之和

为454.求被除数.

解:a?12b?26,a?b?12?26?454,12b?26?b?12?26?454,

(12?1)b?454?12?26?26?390,b=30, 被除数a=12b+26=360+26=386.

这题的后面部分是小学数学的典型问题之一——“和倍” 问题。

2.证明:(1) 当n∈Z且n?9q?r(0?r?9)时,r只可能是0,1,8; 证:把n按被9除的余数分类,即:若n=3k, k∈Z,则n?27k, r=0; 若n=3k +1, k∈Z,则n?(3k)?3(3k)?3(3k)?1?9k(3k?3k?1)?1,r=1; 若n=3k-1, k∈Z,则n?(3k)?3(3k)?3(3k)?1?9(3k?3k?k?1)?8,r=8.

332323322333n3n2n??的值是整数。 (2) 当 n∈Z时,

326n3n2n2n3?3n2?n32??=证 因为,只需证明分子2n?3n?n是6的倍数。 32662n3?3n2?n?n(2n2?3n?1)?(n?1)n(2n?1)

?(n?1)n(n?2?n?1)=n(n?1)(n?2)?(n?1)n(n?1).

由k! 必整除k个连续整数知:6 |n(n?1)(n?2),6 |(n?1)n(n?1). 或证:2!|(n?1)n, (n?1)n必为偶数.故只需证3|(n?1)n(2n?1).

若3|n, 显然3|(n?1)n(2n?1);若n为3k +1, k∈Z,则n-1是3的倍数,得知

(n?1)n(2n?1)为3的倍数;若n为3k-1, k∈Z,则2n-1=2(3k-1)-1=6k-3, 2n-1

是3的倍数.

综上所述,(n?1)n(2n?1)必是6的倍数,故命题得证。

(3) 若n为非负整数,则133|(11n+2+122n+1).

证明:利用11n+2+122n+1=121×11n +12×144 n =133×11n +12×(144 n-11 n)及例5的结论. (4)当m,n,l∈N+时,

(m?n?l)!的值总是整数

m!n!l!证明:(m?n?l)!=(m?n?l)(m?n?l?1)(n?l?1)(n?l)(n?l?1)(l?1)?l!

由k!必整除k个连续整数知:m!|(m?n?l)(m?n?l?1) n! |(n?l)(n?l?1)(n?l?1),

(l?1),从而由和的整除性即证得命题。

(5)当a,b∈Z且a ≠-b,n是双数时,?a?b?|(an?bn); (6)当a,b∈Z且a ≠-b,n是单数时,?a?b?|(an?bn). 解:利用例5结论:若a ≠ b,则?a?b?|(an?bn).令b=-b*, 即得。 或解: a = (a+b)-b, (5) 当n为双数时,由二项式展开

n an?bn??a?b?b?b?????n??a?b??n?a?b?nn?1b????1?n?1证得。(6) 当n为单数时类似可得。 n?a?b?bn?1,

3.已知a1,a2,a3,a4,a5,b∈Z,且

?ai?152i?b2,说明这六个数不能都是奇数.

解:若这六个数都是奇数,设ai?2ki?1,ki?Z,i?1,2,3,4,5,则

?a??(2k?1)2iii?1i?1552?4?ki(ki?1)?5,因为2|ki(ki?1),所以8 | 4?ki(ki?1),

i?1i?155?ai?152i?8q?5,q?Z, 而b2?(2k?1)2?4k(k?1)?1,b2?8q*?1,k,q*?Z,

即等式左边被8除余5, 而右边被8除余1, 故不可能这六个数都是奇数。

4.能否在下式的各□内填入加号或减号,使下式成立;能的话给出一种填法,否则,说明理由。

1□2□3□4□5□6□7□8□9=10

不能,因为等式左边有单数个单数,它们的和差只能是奇数,而等式右边10为偶数。或解:无论各□内填入加号或减号,1□2□3□4□5□6□7□8□9+1+2+3+4+5+6+7+8+9总是偶数,而1+2+3+4+5+6+7+8+9=45,因此的结果1□2□3□4□5□6□7□8□9一定是奇数。 5.已知:a,b,c均为奇数.证明ax?bx?c?0无有理根。 证:若有有理根,记为

2ppp,p,q互质,代入方程有a()2?b??c?0

qqq即ap?bpq?cq?0,这是不可能的,因为p,q互质,二者不可能同时为偶数。 若p为偶数,则ap?bpq为偶数,但cq是奇数,它们的和不可能为0; 若q为偶数,则bpq?cq为偶数,但ap是奇数,它们的和也不可能为0。

222222

6.在黑板上写出三个整数,然后擦去一个,换成其他两数之和加1,继续这样操作下去,最后得到三个数为35,47,83.问原来所写的三个数能否是2,4,6?

解:不能.因为原来所写的三个数若是2,4,6,每次操作后剩下的三个数是两偶一奇.

7.将1-—99这99个自然数依次写成一排,得一多位数A=1 2 3 4 5 6 7 8 9 1011…97 98 99,求A除以2或5、4或25、8或125、3或9、11的余数分别是多少?

解:由数的整除特征,2和5 看末位,∴ A除以2余1,A除以5余4;4和25 看末两位,∴ A除以4余3,A除以25余24;8和125看末三位,∴ A除以8余3,且除以125余24;3和9看各位数字的和,1+2+3+4+5+6+7+8+9=45,A所有数字的和等于450, ∴ A除以3和9都余0,A除以11的余数利用定理1. 4, 计算奇数位数字之和-A 的偶数位数字之和.奇数位数字之和1+3+5+7+9+(0+1+…+9) ×9,偶数位数字之和2+4+6+8+(1+2+…+9) ×10,两者之差为-40,原数除以11的余数就是-40除以11的余数:4.

8.四位数7x2y能同时被2,3,5整除,求这样的四位数.

解:同时被2,5整除,个位为0,再考虑被3整除,有4个:7020,7320,7620,7920. 9.从5, 6, 7, 8, 9这五个数字中选出四个不同的数字组成一个四位数,它能同时被3, 5, 7整除,那么这些四位数中最大的一个是多少?

被5整除,个位必为5. 5+6+7+8=26, 5+6+7+9=27 ,5+6+8+9=28,5+7+8+9=29中唯27能被3整除,故选出的四个不同的数字是5, 6, 7,9,但不同排序有9765,9675,7965,7695,6975,6795,

从最大的开始试除,得9765=7×1395,那么要求的就是9765了。 10.

11.1至1001各数按以下的格式排列成表,像表中所示的那样用—个正方形框住其中的9个数,要使9个数的和等于(1)2001,(2)2529,(3)1989,能否办到?如能办到,写出框里的最小数与最大数.如办不到,说明理由.

181522291623310172441118255121926613202771421 2899599699799899910001001解:设框里居中心的数为x,则9个数的和等于9x. (1) 9不能整除2001,∴和等于2001办不到;(2) 9x=2529,x=281,是所在行第一个数,∴和等于2529办不到;(3) 9x=1989,x=221,和等于1989能办到,框里的最大数为x+8=229,最小数为x-8=213. 12.证明:7(或11或13) |anan?1a3a2a1a0的特征是:7(或11或13) 整除

|anan?1a3?a2aa| 10解答:因为7×11×13=1001。(谐“一千零一夜”) ∴anan-1…a3a2a1a0=7×11×13×a2a1a0+(anan-1…a3-a2a1a0) ×1000.

附)广西师范大学 赵继源主编的《初等数论》习题1—1中的部分题目

3.已知a,b,c中,有一个是2001,有一个是2002,有一个是2003,试判断(a—1)×(b—2)×(c—3)的奇偶性,并说明理由. 6.24|62742??,求?,?.

9. 是否存在自然数a和b,使a2-b2 = 2002成立? 11.证明:当n∈Z时,6 | n(n+1)(2n+1).

12.已知:f?x??ax2?bx?c,f (0),f (-1),f (1),x均为整数.证明:f?x??Z. 解答:

3.偶数.因为a,b,c中,有三个奇数,所以a-1,c-3中至少有一个是偶数. 6.只需3|62742??,且8|62742??,即3|(???),且8|??,先考虑??0,2,4,6,8,有5组解 ????0,???2,???4,???7,???9, ???????0;???4;???8;???2;???6.9.不存在.利用a2-b2 =(a-b)(a + b),而a-b,a + b的奇偶性相同.而2002=2×1001. 11.用数学归纳法或n(n+1)(2n+1)= n(n+1)(n+2)+(n-1)n(n+1),利用整除的基本性质(13). 12.由f (0),f (-1),f (1),x均为整数可得c, a+b, a-b均为整数. 进而知2a,2b为整数.

分类讨论(k∈Z): x=2k时,由2a,2b为整数f (x)显然为整数;

x=2k+1时,f (2k+1) = 4ak(k+1) + 2bk + a + b + c, 可知f (x)仍然为整数。

习题1-2

1. 判断下列各数中哪些是质数?109,2003,17357

2. 求证:对任意 n∈Z+,必有 n 个连续的自然数都是合数. 3. 当 n 是什么整数时,n4+ n2+1是质数? 4. 求证:当 n∈Z+时,4n3+6n2+4n +1是合数. 5. 求 a,使 a,a +4,a +14都是质数.

6. 已知两个质数 p和 q满足关系式 3p+5q=31.求p/(3q+1)的值. 7. 已知 p>3,且 p和 2p+1都是质数,问 4p+1是质数还是合数? 8. 由超级计算机运算得到的结果(2859433-1)是一个质数,试问:(2859433+1)是质数还是合数?请说明理由.

9. 已知:质数 p、q使得表达式(2p+1)/q及(2q-3)/p都是自然数,求 p、q的值 . 10. 试证:形如 4n -1的数中包含有无穷多个质数 . 11.(1)若 n 是合数,证明:2n-1也是合数;(2)有人认为下列各和数:1+2+4,1+2+4+8,

1+2+4+8+16,…交替为质数与合数,你认为对吗? 12. 已知:质数 p≥ 5,且是质数,证明:4p+1必是合数 .

习题1-2解答

1.109?11, 109用质数试除到7,

2003?45,2003用质数试除到37,可知两者是质数,

17357=17×1021是合数. 试除时,用数的整除特征考虑:2,3,5显然不能整除它,由上节第8题结论,357-17= 340,340不能被7,11,13整除,再用17考虑,得分解式。 2. 为作一般性证明,可如下构造 n 个连续自然数:(n + 1)!+ 2,(n + 1)!+ 3,…,(n+ 1)!+ n + 1显然它们每个都是合数.

3. 利用 n4+ n2+1 = n4+ 2n2+1-n2 = (n2+ n+ 1)(n2-n+ 1),知仅当 n= ± 1时,n4+ n2+1为质数 . 4. 利用4n3+ 6n2+4n+1= (2n+1)(2n2+ 2n+ 1) ,n∈Z+, n≥1,2n+1和2n2+ 2n+ 1皆为大于1的数. 5. a=3. 思路:分类讨论(k∈Z): ∵ a=3k+1时,a + 14是3的倍数,a=3k+2时,a + 4是3

的倍数。∴ 必有a =3k,即a为3的倍数。而a是质数, 只有a =3时,三个数全是质数。 6. 条件为一个不定方程, 可知1 < q≤ 5, 穷举得q=2, p=7; q=5,p=2两组解。故1或 1/8。 7. 合数. 利用质数 p> 3得 p不是 3的倍数,p= 3k+ 1,3 | 2p+1,所以,p=3k+2,3 | 4p+1. 或解:4p,4p+1,4p+2是三个连续整数,必有一个被3整除,由题设,只有3 | 4p+1. 8. 合数 . 2859433不可能是3的倍数,连续三个自然数中必有一个是 3的倍数. 即(2859433+1)。另一种解法:由习题1—1第1题(2)的结论,(2+1)|(2859433+1).

9. 设2p?1?h,2q?3?k,h、 k 必为奇数, 2p?1?4p?2?4p?2?4p?2,得k?4,而k

qpq2qkp?3kp不能为3, 故只有k =1, 这样2q-3=p , 代入h?4q?5?4,同时质数 p、q 大于 3. 所以, 只

q能有h =3, 因而得 q =5, p=7.

10. 先证:一切大于 2的质数,不是形如 4n + 1就是形如 4n-1的数;再证任意多个形如 4n+1的数,最后用数学归纳法验证 .

若形如 4n-1的质数只有有限个:p1, p2, …, pk。令N = 4 p1 p2 … pk-1,N为形如 4n-1的数,由假设N必为合数,且必有一个形如 4n-1的质因数p(为什么?), 因此p为 p1, p2, …, pk中在某一个,于是,p | 1, 矛盾。 11.(1)n是合数, 设n=st, 2n-1=2st-1=(2s-1)[(2s)t-1+ (2s)t-2+ … + 2s+ 1]. (2)1+2+22+ … +2n-1=2n-1. 当 n=14,15时,214-1,215-1均为合数,∴ 不对 . 12. 书后提示说取模为6分类讨论 p,即设 p=6q+ r(r=0,1,2,3,4,5).

由质数 p≥ 5,若p=6q, 6q+2, 6q+3或6q+4, p皆为合数, 不可能. 若p=6q+ 1,

则2p +1=12q+ 3也是合数, 故在题设条件下, 只有p=6q+5, 此时4p+1=24q+21, 是合数. 实际上,这题与第7题完全相同。质数p> 3?质数p≥ 5,可用前面的方法简单求解。 习题 1-3 1.求:(1)(21n +4,14n +3)(其中 n∈Z+); (2)(30,45,84),[30,45,84];(3)(5767,4453). 2.求证:[an,bn]= [a,b]n(a,b,n∈Z+).

3.自然数 N =10x+ y(x是非负整数,y是 N 的个位数字),求证:13 N的充要条件是 13 (x+4y).

4.用割(尾)减法判断下列各数能否被 31,41,51整除:26691,1076537,1361241

5.有 15 位同学,每位同学都有编号,他们是 1 号到 15 号 .1 号同学写了一个自然数,2号说“这个数能被 2整除”,3号说“这个数能被 3整除”……依此下去,每位同学都说这个数能被他的编号整除 .1 号做了一一验证,只有编号连续的两位同学说的不对,其余同学都对 .问:(1)说得不对的两位同学的编号是什么数?(2)如果 1号写的数是 5位数,这个 5位数是多少?

6.请填出下面购物表格中□内的数字: 品名 课桌 课椅 数量 72 77 单价(元) □.□□ □.□□ 总价(元) □□7.7□ 3□□.□□ □□3□.55 合计金额(元) 7. 狐狸和黄鼠狼进行跳跃比赛,狐狸每次跳 4 12 米,黄鼠狼每次跳 2 34米,它们每秒钟都只跳一次,比赛途中,从起点开始,每隔 12 38米设有一个陷阱,当它们之中有一个掉进陷阱时,另一个跳了多少米?

8. 大雪后的一天,大亮和爸爸共同步测一个圆形花圃的周长,他俩的起点和走的方向完全相同,大亮每步长 54厘米,爸爸每步长 72厘米 .由于两人脚印有重合,所以雪地上只留

下 60 个脚印,求花圃的周长 . 9. 设 a,b是自然数,a + b=33,[a,b]=90,求(a,b).

10. 一公路由 A 经 B 到 C,已知 A、B 相距 280 米,B、C 相距 315米,现在路边植树,要求相邻两树间的距离相等,并要求在 B 点、AB、BC 的中点上都要植上一棵树,那么两树间的距离最多有多少米?

11. 一袋糖不足 60 块,如果把它平均分给几个孩子,则每人恰好分得 6块;如果只分给这几个孩子中的男孩,则每个男孩恰好分得 10块 .这几个孩子中有几个女孩?

12. 爷爷对小明说:“我现在的年龄是你的 7 倍,过几年是你的 6倍,再过若干年就分别是你的 5 倍、4 倍、3 倍、2 倍 .”你知道爷爷和小明现在的年龄吗?

习题 1-3解答

1.(1)1. 用辗转相除法 (2)1260. (3)73. 用辗转相除法

nn?an??anbbnn2. 证: 由定理1.14?a,b?????a,b?,?a,b????,n?nn??a,b?n??a,ba,ba,b?????????nn?n ??a,b?,

????a?n?b?n?ab??而由定理1.13, ,从而由定理1.21推论3,?=1。 ,??a,b?,?a,b???1???a,b????a,b????????????∴(an,bn)=(a,b)n,再由定理1.19,[a,b](a,b)= a b,等式两边同时n次方,得

[a,b]n(a,b)n = a n b n, 同样由定理1.19, [an,bn](an,bn)= an bn, ∴ [a,b]n(a,b)n =[an,bn](an,bn); ∴ [a,b]n =[an,bn]。 3. 利用10x+ y= 10(x+4y)-39y.

4. 31| 26691,41|26691,51|26691;31|1076537,41|1076537,51|1076537;31|1361241,41|1361241,51|1361241.

以51为例,51|26691?51|(2669-1×5);又51|2664 ?51|(266-4×5);显然51|246 。51|1361241?51|(136124-1×5),又51|136119?51|(13611-9×5),又51|13566?51|(1356-6×5),又51|1326?51|(132-6×5),而51|102。

5. (1)这两个连续的编号的倍数应该大于15, 否则编号是它们的倍数的同学说的也不对; 而且是这两个连续的编号的质因数的次数应该高于比它小的数,否则编号是它们的质因数的同学中至少也有一个说的也不对。因此只能是8,9.

(2)60060;因为1号写的数是2到15除8,9之外的整数的公倍数,也就是3,4,5,7,11,13的公倍数,3,4,5,7,11,13两两互质,它们的最小公倍数60060就是5位数。 6. 72=8×9,8,9互质,故总价必为8,9的倍数,可推得为 707.76元,因而知课桌的单价为9.83元;课椅的总价为 3□□.79元,由77=7×11推得另两个数字,即课椅总价为 328.79元,再得课椅单价为 4.27 元;合计金额为 1036.55元 .

7. ?4500,12375??49500,?2750,12375??24750,24750较小,24750?2750?9. 黄鼠狼在第9跳掉进陷阱,此时狐狸跳了4.5×9 = 40.5米 . 8. [54,72]=216,每216厘米有脚印

216216??1?6个,故花圃的周长2160厘米 . 54729. 此题应该先讨论a + b,[a,b]与(a,b)的关系。

令(a,b)?d,a?dt1,b?dt2,?t1,t2??1,?x,y使xt1?yt2?1,x(t1?t2)?(y?x)t2?1, ?(t1?t2,t2)?1.同理(t1?t2,t1)?1.?(t1?t2,t1t2)?(t1?t2,t1)?1(定理1.21),

?(dt1?dt2,dt1t2)?d(定理1.14).即,?a,b??(a?b,?a,b?). ( 33, 90 ) = 3, 所以 ( a, b ) = 3.

10. 因为AB、BC 的中点上都要植上一棵树,315÷2=157.5因此应考虑1400和1575的最大公约数175。最后答案:两树间的距离最多有17.5米 . 11. 2个 .

12. 设小明 x岁,则爷爷 7x岁,7x +h =6(x+h) , x=5h; 7x +k =5(x+k) , x=2k; 7x +i =4(x+i) , x=i; 7x +j =2(x+j) , 5x=j; 知小明年龄是2, 5的倍数。因此小明 10岁,爷爷 70岁. 习题 1-4

1.把下列各数分解质因数:2001,26840,111111

2.将 85,87,102,111,124,148,154,230,341,354,413,667分成两组(每组 6个数),怎么分才能使每组各数的乘积相等?

3.要使下面四个数的乘积的最后 4个数字都是 0,括号中最小应填什么自然数?975×935×972×(). 4.用分解质因数法求:(1)(4712,4978,5890);(2)[4712,4978,5890].

5.若 2836,4582,5164,6522 四个数被同一个自然数相除,所得余数相同,求除数和余数各是多少?

6.200以内仅有 10个正约数的自然数有几个?并一一求出 . 7.求:(1)%(180);(2)&(180);(3)&1(180). 8.已知[A,B]=42,[B,C]=66,(A,C)=3,求 A,B,C .

9.一个自然数有 21个正约数,而另一个自然数有 10个正约数,这两个数的标准分解式中仅含有不大于 3的质因数,且这两个数的最大公约数是 18,求此两数是多少?

10.小明有一个三层书架,他的书的五分之一放在第一层,七分之几(这个几记不清了)放在第二层,而第三层有书 303本,问小明共有书多少本?

11.某班同学(50人左右)在王老师带领下去植树,学生恰好能分成人数相等的 3 组,如果老师与学生每人种树的棵数一样多,共种了884棵,那么每人种多少棵树? 12.少年宫游乐厅内悬挂着 200个彩色灯泡,这 200个灯泡按 1耀200编号,它们的亮暗规则是:第 1秒:全部灯泡变亮;第 2秒:凡编号为 2的倍数的灯泡由亮变暗;第 3秒:凡编号为 3的倍数的灯泡改变原来的亮暗状态,即亮的变暗,暗的变亮 .一般地,第 n 秒凡编号为 n 的倍数的灯泡改变原来的亮暗状态。这样继续下去,每 4分钟一个周期,问第 200 秒时,明亮的灯泡有多少个? 习题 1-4解答 1. 2001= 3×23× 29, 26840= 23×5×11× 61, 111111= 3× 7×11×13× 37.

2. —组为:85,111,124,154,354,667;另一组为:87.102,148,230,341,413. 3.20.四个数分解质因数后一共应该有且 且只有4个2与4个5,需补充2个2与1个5。 4.(1)38, (2)3086360.

5.除数为l或2时,余数为0;除数为97时,余数为23;除数为194时,余数为120. 6.有5个,10=2×5=1×10因此所求的数应该为ab或c后者即令c=2也已经超出200,因此分别令a=2.b=3; a=2.b=5; a=2.b=7; a=3,b=2; a=2.b=11; 得48,80,112,162,176. 7.(1)18. (2)546 (3)180'.

8.因为B |?B,C? , B |?A,B?, 所以B是66,42的公约数,因而B是6的约数。又

49

因为?B,C??66?2?3?11,?A,B??42?2?3?7,所以7|A,11|C,从而设

因为知?2??2?1,且?1?1。A?2?13?27,B?2?13?2,C?2?13?211, 由?A,C??3,若B不含2的话,由?B,C??66,∴A?2?1?3?7, ?A,B??42,A,C就必须同时含2, 与?A,C??3矛盾。

B?2?3?2,C?2?1?3?11,?1,?2,?1?1,0,而且?1与?1不能同时为1.

?1?0,?1?三种情况1,共得6组于是?2?1和0时,各有?1?1,?1?0;?1?0,?1?0;解,分别为:

9. 576和162 10.3535本。解:由题目可知小明的书的册数是35的倍数, 设为35k, 可列出方程28k-5xk=(28-5x)k=303=3×101知k=101.

11. 分解质因数:884=4×13×17=17×52=68×13,884的因数中有4, 13, 52都具有3k+1形式,只有52=符合50人左右的题设,因此学生51人。

12. 灯的一次“改变”对应着它的编号的一个因子. 要使灯仍旧亮着需要奇数次“改变”.什么样的数有奇数个因子呢? 由定理1.26公式⑴知只有完全平方数! 200以内的完全平方数只有14个。即为答案. 此题也可先考虑10个灯泡。用归纳得出“只有完全平方数”的结论。 习题1-6部分习题解答 2.

3?77?1?1??3?7?, 代入得10。 2?7?3,?a????2,b??2????22?3?7??2? ?2?试讨论?xy?与?x??y?的大小关系。3. 若x,y?R?,?1?求证:?xy???x??y?;证:?1??xy?????x?+?x????y?+?y?????x??y?????x??y?+?x??y?+?x??y???

?? 显然 ???x??y?+?x??y?+?x??y????0,证得。?2?x,y?Z+时,?x???y???xy??0,有?x??y???xy?;而 x?1.2,y?2.1时,xy?2.52,?xy??0.52,?x??y??0.02,?x??y???xy?; 又 x?1.9,y?1.8时,xy?3.42,?xy??0.42,?x??y??0.72,?x??y???xy?.可见,三种情况都有。 4. 解方程:(1)3x?5?x??50?0 解:?x??50?3x5y?50 是整数,设其为y.由原方程得x?5350?3x8x?508x?505055又?x??x??,0??x???1.即0?8x?50?5,?x?.

5558820505y?505561. 亦即??,13??y?14?, y?14 ?x?383888

(2)原式化为?x??x(x??x?)?0,整理后再由一元二次方程求根公式得 ?x??25?15?15?1x,与相乘后的积为整数,只能是x?。 2225. 15< x+ y<16.

6. 25!=222×310× 56× 73×112×13×17×19×23. 由定理1.29公式求出各个质数的指数。 7.(1)??199?1??199?2???????97??97??199?96??? ??97?5?1??5?2?5?96??解: 原式??2?1? ?2?2???2?96???????97??97?97????5?1??5?2??5?96??2?1?2?2??2?96???????97????97???97??

?5?1??5?19??5?20??9312???????????979797???????5?78??5?96????????97???97??=9312+0+…+0+19+40+57+76=9504.

(2)考虑1??5?38??5?39????????9797?????5?58????97??11?2232?111?1??200821?22?3?1

2007?2008?11??11??1??????????12??23?1??1?????20072008??1?=1.

20082???1?1?8. 1373个 .

9. 14人 . 10. 49盏 .

11?2232?1111?1??2。从而?2??22?23200822008?x??1??211. 高中时我们已知x?1?x?1???2x?1?x?1,

?2x?1?x??1时,?x???2,??2?x??1;

?1?x?1时,?x??2x,即??x??0??1?x?0,即?2?2x?0,而2x?Z,x?1时,?x??2,?2?x?3.?x??,x?0;

∴ -2≤ x<-1或 2≤ x<3或 x= -1/2 或 x= 0.

12. 解: 等式左边为73个数相加,而546?73?7?35.?7?x?8,

12

且可知等式左边从右向左有且只有满足x?由等式左边从右向左第35项x?y?8 10057?8移项得x?7.43. 100??100x??743.

习题2-1

5. 若69, 90和125关于某数 d 同余, 证明对于d, 81与 4同余. 证明:由69和90关于 d 同余, d | 90- 69, d | 21,

90和125关于某数 d 同余, d | 125- 90, d | 35, ∴ d | (21, 35) , d=1或7. 22

9. 由 (n, 8)=1可知,n为奇数. 设n=2k+1, n-1= 4k (k+1),8 | (n-1).

12. 4+1=5, 因此个位为4的2n, 加1后都能被5整除. 先考察n=1, 2, … , n 较小的情况:

2,4,8,16,32,64,128,256,512,1024,2048,个位为6的幂间隔4次得重复出现, 又6 ×4=24. 因此?24??4?1?24k?2?1能被5整除.

k

14.

+

即n=4k+2(k∈Z).

任意平方数的末位数字都不能是 2, 3, 7, 8的某一个.

2 22

证:令a=(10x+y), 则a2=(10x+y)≡y (mod 10). 令=0,1…9, y 的个位不能是2, 3, 7, 8.

2

因此,数字 a (1≤a≤9) 的平方 a的末位数字也没有2, 3, 7, 8. 习题2-2

3. 3?3?242?11?22.3?3?350nmm?3n?m?1??

4. 偶数m的最小非负完全剩余系中奇偶各半.任一组完全剩余系中各数必与0,1, …m-1中一个数同余,故均可写成mkr+r,r= 0,1, …m-1的形式.故亦奇偶各半. 其他的都是较基本的题目, 请看书后的答案或提示. 习题2-3

1.乘幂 20,21,22,…,29能否构成模 11的一个简化剩余系? 解:i > j时,2-2=2(2

i

j

j

i-j

-1), 11|2, 通过验证可知,对任何i,j,也有11|(2

j

i-j

-1),

φ (11) = 10,而20,21,22,…,29为10个不同的整数,所以它们构成模 11的一个简化剩余系

2.列表求出模 m 为 10,11,12,…,18等值时的最小简化剩余系及相应的φ (m). m 10 11 12 1 1 1 2 3 3 4 5 5 6 最 小 简 化 剩 余 系 7 7 7 8 9 9 10 11 φ (m) 4 10 4

13 14 15 16 17 18 1 1 1 1 1 1 2 2 2 3 3 3 3 4 4 4 5 5 5 5 5 6 6 7 7 7 7 7 8 8 8 9 9 9 9 10 11 12 11 11 11 11 13 13 13 15 17 12 6 8 8 16 6 13 14 10 11 12 13 14 15 16 3.证明定理 2.7.

证明:(必要性)∵ x1,x2,…,xk是模 m 的简化剩余系,

∴ k=φ(m),且当 i ≠ j时,xi?xj(mod m),(xi,m)=1,i = 1,2,…,φ(m). (充分性)k=φ(m),∴ x1,x2,…,xk共有φ(m)个.

又 xi?xj(mod m),(i ≠ j,1≤i,j≤ k),(xi,m)=1(i=1,2,…,k),

∴ x1,x2,…,xk各属于φ(m) 个不同的且与 m 互质的剩余类, ∴ x1,x2,…,xk是模 m 的简化剩余系. 4. 验证:(1)8,16,24,32,40,48是模 7的简化剩余系;

(2)11,13,77,99是模 10的简化剩余系. 解:(1)∵(4,7)=1,可化为2,4,6,8,5,12,又5≡12(mod 7),

∴ 8,16,24,32,40,48不是模 7的简化剩余系。

(2)10的最小简化剩余系是1, 3, 7, 9。11,13,77,99分别与1, 3, 7, 9关于模10同余。∴ 11,13,77,99是模 10的简化剩余系. 5. 当 m 取下列各值时,计算φ(m)的值 .

25,32,40,48,60,120,100,200,4200,9450.

答案:φ(25)= 20,φ(32)=16,φ(40)=16,φ(48)= 16,φ(60)=16,φ(120)= 32,

φ(100)= 40,φ(200)= 80,φ(4200)= 960,φ(9450)= 2160.

6. 若φ(m)是奇数,试求 m 的值. 解:(参看下一题) m = 1或 m =2. 7. 当 m >2时,证明φ(m)是偶数 .

证:设 m = p1α1p2α2… pkαk,∵ m >2,∴ 至少存在 i,αi> 1或存在 j,pj是奇数, ∴ p1α1- p1α1 -1,…,pkαk- pkαk-1中至少有一个为偶数,知 φ(m)必为偶数 或证:若有?i?1,无论pi为奇为偶,有pi?i?pi?i?1=pi?i?1?pi?1?是偶数.

所有的?i?1,则必有一个pi为奇数.因为如果m为质数,?(m)?m?1,大于2的质数是奇数,m?1是偶数;如果m为合数,pi中最多只有一个是2(偶数),那么至少有一个为奇数,这时?(m)???pi?1?,显然是偶数.

i?1k

8. 试证:使φ(m) =14的数 m 不存在.

证:φ(m) =14=2×7= p1α1 -1…pkαk-1 (p1-1)…(pk-1),2,7是质数,所以必有p1=2,p1=7,这是不可能的。

9. 已知φ(m) = 4,求 m .

解:设m = p1α1p2α2… pkαk,由φ(m)= (p1α1- p1α1 -1)…(pkαk- pkαk-1),φ(m) = 4=4×1=22,得 m = 5,φ(m) =5-1= 4,或 m =8=23,φ(m) = 22或 m = 10=5×2,φ(m) =4×1,或 m =12.

10. 如果 n =2m,(2,m)=1,那么φ(n)= φ(m). 11. 若 m 是奇数,则φ(4m)=2φ(m).

12.(1)分母是正整数 n 的既约真分数的个数是多少?为什么? (2)分母不大于 n 的既约真分数的个数是多少?为什么?

解 10.∵(2,m)= 1,∴ φ(n)=φ(2m)=φ(2)φ(m)=φ(m). 11. ∵ m 是奇数,∴(4,m)= 1,则φ(4m)=φ(4)φ(m).

∵ φ(4)= 2,∴ φ(4m)=2φ(m).

12.(1)φ(n). (2)φ(2)+φ(3)+ … +φ(n). 习题 2-4

1.举例说明欧拉定理中(a,m)=1是不可缺少的条件 . 2.求下列各题的非负最小余数:(1)84965除以13; (2)541347除以17;

(3)477385除以19; (4)7891432除以18; (5)(127156+34)28除以111. 解答:1. 当 a= 2,m =4时,?(4) =2,此时 22≡0(mod 4),可见(a,m)= 1是欧拉定理的不可缺少的条件 .

2.(1)8. (2)10. (3)16. (4)1. (5)70. (1)84965除以13;(13,8)=1, ∴ 812≡1(mod 13),84965=(812)413×89≡1×(-1)4 ×8(mod 13) 或解:82≡-1(mod 13),84965=(82)2482×8 ≡ (-1) 2482 ×8 ≡ 8(mod 13)。 3.设 p,q是两个大于 3的质数,求证:p2≡ q2(mod 24). 4.设 p是大于 5的质数,求证:p4≡1(mod 240). 解答:3. 24=3×8,(3,8)= 1. 由条件,( p,3 ) = ( q,3 ) = 1,由费尔马小定理有 p2≡1(mod 3), q2≡1(mod3). ∴ p2 ≡ q2(mod 3). 又 ∵ p,q 必为奇数,由习题2-1第9题的结论,有p2≡1(mod 8),q2≡1(mod8). ∴ p2 ≡ q2(mod 8). ∴ p2 ≡ q2(mod 24).

2

4. 240 = 3×5×16,由条件,( p,3 ) = ( p,5 ) = 1,∴ p4≡1(mod5),p4≡(p2)≡1(mod3). 又 p为奇质数,从而 2 |(p2+ 1),8 |(p2-1),∴ 16 |(p4-1),即 p4≡1(mod 16). 而(3,5)=(3,16)=(5,16)= 1. ∴ p4≡1(mod 240). 5. 已知 p是质数,(a,p)=1,求证:(1)当 a 是奇数时,ap-1+(p-1)a ≡ 0(mod p); (2)当 a 是偶数时,ap-1-(p-1)a ≡ 0(mod p).

6. 已知 p,q 是 两 相 异 的 质 数,且 ap-1≡1(mod q),aq-1≡1(mod p), 求证:apq≡ a(mod pq). 解答:5.(1)由 p 是 质 数,(a,p)= 1,a 为 奇 数,有 ap-1≡ 1(mod p).

(p-1)a ≡-1(mod p),∴ ap-1+ (p-1)a≡ 1-1≡ 0(mod p). (2)由条件,ap-1≡1(mod p), (p-1)a≡1(mod p),∴ap-1-(p-1)

?(n)≡1-1≡0(mod p).

6. ∵ ap ≡ a(mod p),∴ apq ≡ (ap)q ≡ aq ≡ a(mod p);同理,apq ≡ (aq)p ≡ ap ≡ a(mod q), 而(p,q)= 1,故 apq≡ a(mod pq). 7. 如果(a,m)=1,x≡ ba

?(m)?1(mod m),那么 ax≡ b(mod m).

8. 设 A 是十进制数 44444444的各位数字之和,B 又是 A 的各位数字之和,求 B 的各位

数字之和 .

9. 当 x∈Z 时,求证:(1)2730 | x13- x;(2)24 | x(x+2)(25x2-1). 解答:7. ∵ x≡ba

?(m)?1(mod m),∴ ax≡ aba

?(m)?1≡ a

?(m)b (mod m). ∵ (a,m) = 1,a

?(m)= 1

(mod m),∴ ax≡ b(mod m).

8. 设 B 的各位数字之和为 C,∵ lg44444444= 4444lg4444 < 4444×4= 17776,即44444444 的位数小于17776,∴ A ≤ 9×17776 = 159984,B < 1 + 9×5 = 46,C ≤ 4 + 6 = 10. 又 ∵(7,9)= 1,?(9) = 6,4444= 6×740+4,44444444 ≡ 7 4444 ≡ 74 ≡ (-2)4 ≡ 7(mod 9),∴ B 的各位数字之和为 7.

9.(1)∵ 2730=2×3× 5× 7× 13,2,3,5,7,13两两互质,x13- x= x(x12- 1), ∴当 2 | x或 2 | x时都有 x(x12-1)≡ 0(mod 2),x(x12- 1)≡ 0(mod 13). 又 ∵x13-x= x(x6- 1)(x6+ 1),∴ 当 7 | x或 7 | x时都有 x(x6- 1)(x6+ 1)≡ 0(mod 7).而x13- x= x(x4- 1)(x8+ x4+ 1),∴ 当 5 | x或 5 | x时,都有 x(x4-1)(x8+ x4+ 1)≡ 0(mod5).又 x13- x= x(x2-1)(x2+ 1)(x8+ x4+ 1),∴ 当 3 | x或 3 | x时,都有x(x2-1)(x2+ 1)(x8+ x4+ 1)≡ 0(mod3). ∴ 2730 | x13- x. (2)解法一,同上。解法二: x(x+2)(25x2-1)= 24 x3(x+2)+ x(x+2)(x2-1), x(x+2)(x2-1)= x(x-1)(x+1)(x+2),四个连续自然数的乘积必能被4!=24整除,证得。10. 设质数 p>3,x∈Z,试证:6p | xp- x.

11. p是不等于 2和 5的质数,k是自然数,证明:p|9999.

?p?1?p个9解答:10. ∵质数 p> 3,∴ (6,p)=1,xp- x= x(xp-1- 1)≡ 0(mod p). 又 p- 1是偶数,∴x(xp-1-1)≡ x(x2- 1)…(mod p). 于是,当 2 | x或 2 | x 时,x(x2- 1)≡ 0(mod 2);当 3 | x或 3 | x时,x(x2-1)≡ 0(mod 3).故 x(xp-1- 1)≡ 0(mod 6).从而6 | p(xp- x). 11. 99(10,p)= 1,∴ 10p-1≡ 1(mod p). 99?10?p?1?k?1. 由条件,

?p?1?k个9∴ (10p-1)k≡ 1(mod p). ∴ p|99?(n)99.

?p?1?p个912. 设(m,n)=1,证明:m证:∵(m,n)= 1,∴n(mod m). 对称可得 m

?(m)+ n

?(m)≡1(mod mn).

?(n)≡1(mod m ),而 m

?(m)x≡ 0(mod m ),∴ m

?(n)?(n)+n

?(m)≡ 1

?(n)+n≡ 1(mod n). ∴ m+n

?(m)≡ 1(mod mn).

13. 已知 a =18,m =77,求使 a≡1(mod m)成立的最小自然数 x. x=30.

满足要求的最小自然数 x必为60 的约数。验算可知。 ?(77)??11?1??7?1??60,由定理,

习题3-1

1.解下列不定方程:

(1)7x-15y=31; (2)11x+15y=7; (3)17x+40y=280; (4)525x+231y=42; (5)764x+631y=527; (6)133x-105y=217. 解:(1)辗转相除得15=7×2+1, ∴ 1 = 15-7×2= 7×(-2)-15×(-1), ∴ 因此原方程的一个解是 x0=-2×31=-62, y0=-1×31=-31;

?x??62?15t原方程的通解为?这里t为任意常数.

?y??31?7t(2)辗转相除得15=11×1+4, 11=4×2+3, 4=3+1 ∴ 1 = 4-3=4-(11-4×2)= 4×3-11=

(15-11×1) ×3-11=15×3 + 11×(-4),

∴ 因此原方程的一个解是 x0=-4×7=-28, y0=3×7=21;

?x??28?15t原方程的通解为?这里t为任意常数.

?y?21?11t(3)用分离整数法:x?280?40y8?6y?16?2y?.

1717观察可知y =-10时,x = 36 + 4= 40.

?x?40?40t∴ 原方程的通解为?这里t为任意常数.

?y??10?17t2. 解下列不定方程:(1)8x-18y+10z=16; (2)4x-9y+5z=8; (3)39x-24y+9z=78;

(4)4x+10y+14z+6t=20; (5)7x-5y+4z-3t=51.

3. 解下列不定方程组:(1) x+2y+3z=10, (2) 5x+7y+3z=25,

x-2y+5z=4; 3x- y-6z=2;

(3) 4x-10y+ z=6, (4) 10x+7y+ z=84,

x-4y- z=5; x-14y+ z= -60;

4. 求下列不定方程的正整数解:(1)5x-14y=11; (2)4x+7y=41; (3)3x+2y+8z=21. 5. 21世纪有这样的年份,这个年份减去 22 等于它各个数字和的495倍,求这年份.

6. 设大物三值七,中物三值五,小物三值二,共物一百三十八,共值一百三十八,问物大中小各几何?

7. 买2元6角钱的东西,要用1元、5角、2角、1角的四种钱币去付,若每种钱币都得用,则共有多少种付法?

8. 把 239分成两个正整数之和,一个数必是 17 的倍数,另一个数必是 24的倍数,求这两位数.

9. 一个两位数,各位数字和的 5倍比原来大 10,求这个两位数.

10. 某人 1981年时的年龄恰好等于他出生那一年的年号的各位数字之和,这个人是在哪一年出生的?

11. 一个四位数,它的个位数上数比十位数字多 2,且此数与将其数字首尾颠倒过来所得的四位数之和为 11770,求此四位数 . 习题 3-2

1.求 x2+ y2= z2中 0< z<60的所有互质的解.

2.求三个整数 x,y,z(x> y> z>0),使 x- y,y- z,x- z都是平方数 . 1.

b a x y z 1 2 3 4 5 1 4 8 1 6 2 3 2 5 2 7 3 4 4 5 9 15 35 5 21 45 7 12 12 20 28 24 40 17 37 13 29 53 25 41 2. 设 x- y= a2,y- z= b2,x- z= c2,则 a2+ b2= c2,因此给出 a,b的值即可求得x,y,z. 3.已知直角三角形斜边与一直角边的差为 9,三边的长互质且和小于 88,求此直角三角形的三边的长 .

4.试证:不定方程 x4-4y4= z2没有正整数解 .

3. 设直角三角形的三边的长为x, y, z. 则由定理,x=a2-b2, y=2ab, z=a2+b2, 由题目得

a2+b2-(a2-b2)=9或a2+b2-2ab=9, 前者无整数解,后者(a-b)2=9, a-b=3. a=4,b=1,则 x=15,y=8,z= 17或a=5,b=2,则x= 21,y= 20,z= 29. a=7,b=4, 则三边的长的和大于88。 4. 因为 z4= (x4-4y4)2 = x8-8x4y4+ 16y8= (x4+ 4y4)-(2xy)4,即(2xy)4+ z4=(x4+4y4)2,就是说,

2

如果x4-4y4= z2有正整数解,则u4+v4= w2有正整数解,与已证定理矛盾,故无正整数解 . 5.试证:每个正整数 n 都可以写为n = x2+ y2- z2,这里 x,y,z都是整数 . 6.求方程 x2-dy2= 1,当 d = 0、d = -1、d < -1 时的非负整数解 . 7.试证:2x2+ y2+3z2=10t2无正整数解 .

5. 适当取正整数 x,使 n - x2= m 为一正奇数,设 y = m + 12 ,因为 y2- m =m -1()22= z2,得 n- x2= y- z2.

2

6. 当 d= 0时,x=1,y为任意非负整数;当 d= -1时,x= 1,y=0和 x= 0,y= 1;当 d< - 1时,x= 1,y=0.

7. ∵y2+ 3z2是偶数,∴y与 z必同奇同偶 .若 y 与 z同为奇数,则 2x2+ y2+3z2被 8除和 10t2被 8 除的余数不相等,故 y 与 z一定同为偶数 .令 y= 2y1,z=2z1,代入原式得,x2+ 2y21+ 6z21= 5t,同样,x 和 t同奇同偶,也同样排除 x 和 t同奇,令 x= 2x1,t= 2t1,代入得,2x21+ y21+ 3z21= 10t21,由于 0< t1< t,矛盾,从而得证 . 习题 3-3

1. 求不定方程 4x2-4xy-3y2=21的正整数解 . 2. 求不定方程 x2+ y2=170的正整数解 . 3. 求不定方程 x2-18xy+35=0的正整数解 . 4. 求 4x2-2xy-12x+5y+11=0的正整数解 . 5. 求 x2+ xy-6=0的正整数解 . 6. 求 y- (x+3y)/(x+2) =1的正整数解 .

7. 设 n =7(mod 8),则 n 不能表示为 3个平方数的和 . 1. 由4x2-4xy-3y2= 21,得(2x+ y)(2x-3y)= 21. 得 2x+ y= 21, 2x+ y=7, 即 x= 8, x= 3, 2x- 3y=1, 2x- 3y= 3. y= 5, y= 1. 2. 由 x2+ y2=170知,x,y同为奇数或同为偶数.

x,y为偶数,则 x2+ y2有因数 4,而 170无 4因数;

x,y为奇数,设x =2k+1, y = 2h+1, 代入化简得k (k+1)+h (h+1) = 42, 仅当k = 0, h = 6或k = 0, h = 6时可求得:

?x?1,?x?13,?x?7,?x?11, ?????y?13;?y?1;?y?11;?y?7.x?35,x是 35的约数,得 x= 1,y= 2,或x=35,y=2. x64. 由原方程变为:y= 2x-1+ ,2x-5是 6的约数:± 1,±2,±3,±6,通过分析得

2x?53. x2-18xy+ 35=0,得 18y=

x=3,y=11或x=4,y=9.

5. x=1,y=51或 x= 2,y= 11.

6. 原方程变形为 y=2+ 4x- 1,可求得 x= 2,3,5,代入可求 y.

7. x2+ y2+ z2= n=7(mod 8),则 x,y,z必有一奇数 .由 x2≡1(mod 8),有 y2+ z2=6(mod 8),即y,z同奇同偶,同奇不成立,同为偶时,由 y≡4(mod8)产生矛盾 . 8. x2+ y2= p≡3(mod4),当 x,y≡0,± 1,2(mod4)时,x2+ y2≡0,1,2(mod 4)),这产生矛盾,命题得证 .

9. 由原方程组中 x+ y+ z=0得 z= - (x+ y),代入 x3+ y3+ z3= -18,则 xy(x+y)=6,故 xyz= -6,x、y、z都是 6的约数,并且只有一个是负数,可得其整数解 x= -3,y= 2,z= 1. 10. 通过证明 x2+ y2+ z2被 8整除所得的余数不等于 - 1即可 . 11. 通过证明 x3+ y3+ z3被 9整除所得的余数不等于 4即可 . 习题4-2(P138)

1.试写出三个模数是18的一次同余式,分别使它有唯一解,无解,有四个解。 2. 下列同余方程是否有解?为什么?如果有解,有多少个解? (1)8x+5≡0(mod 23);(2)15x+7≡0(mod 12);

(3)34x≡0(mod 51);(4)30x≡18(mod 114);(5)174x≡65(mod 1309). 3.用同解变形法解下列同余方程:(1)3x≡2(mod 7);(2)9x≡12(mod 15); (3)15x≡9(mod 6); (4)20x≡44(mod 72); (5)40x-191≡0(mod 6191);(6)256x≡179(mod 337). 4.用化为不定方程的方法解下列同余方程:

(1)20x≡4(mod 30); (2)64x≡83(mod 105); (3)57x≡87(mod 105); (4)4x≡11(mod 15); (5)47x≡89(mod 111); (6)10x≡22(mod 36). 5.利用欧拉定理解下列同余方程:

(1)6x≡22(mod 36); (2)3x≡10(mod 29); (3)258x≡131(mod 348); (4)11x≡7(mod 13); (5)3x≡2(mod 17); (6)243x≡102(mod 551). 6.用求组合数的方法解下列同余方程:

(1)5x≡13(mod 43); (2)9x≡4(mod 2401); 习题4-3(P154)

第2题a取什么值时,下面的同余方程组有解?

解:因为(18, 21)=3, 3|8-5,所以??x?5(mod18),?x?5(mod18),有解;(18, 35)=1, 所以??x?8(mod21).?x?a(mod35).?x?a(mod35),有解。

?x?8(mod21).总是有解。因此,要使题设的同余方程组有解,只需?而这里,(21, 35)=7,由定理可知,只需a≡8≡1(mod 7). 即x = 1+7t ( t为整数 ),题设的同余方程组总有解。 3.解下列同余方程组:(1) ??x?3(mod7),?x?6(mod13),, (2) ?

?x?5(mod11).?x?7(mod24).?2x?4(mod8), ?15x?5(mod35).?(4) ??5x?7(mod11), (5)

6x?9?0(mod19).?解:(1)方法一:设x=5+11y, 代入第一个同余方程,得11y≡3-5 (mod 7),

得y≡3 (mod 7) 所以同余方程组的解是x≡38 (mod 77)。 方法二:用孙子定理解,M=77,M1=11,M2= 7, 令11 M1′≡1(mod 7), 得M1′≡2(mod 7), 7 M2′≡1(mod 11),得M2′≡-3(mod 11),

所以同余方程组的解是x≡11×2×3 + 7×(-3)×5≡-39≡38 (mod 77). (2) 方法一:设x=7+24y, 代入第一个同余方程,得24y≡6-7 (mod 13), 得y≡7 (mod 13) 所以同余方程组的解是x≡7+24×7≡175 (mod 312)。 方法二:用孙子定理解,M=312,M1=24,M2= 13, 令24 M1′≡1(mod 13), 得M1′≡6≡-7 (mod 13), 13 M2′≡1(mod 24),得M2′≡13(mod 24),

所以同余方程组的解是x≡24×(-7)×6 + 13×13×7≡-1008+1183≡175 (mod 312).

第4题(1) (3) (4).

5..解下列同余方程组: (1) x≡8(mod 15), (2) x≡6(mod 11),

x≡3(mod 10), x≡3(mod 8), x≡1(mod 8); x≡11(mod 20); (3) x≡2(mod 35), (4) 4x≡90(mod 105),

x≡9(mod 14), 5x≡18(mod 63), x≡7(mod 20); 7x≡10(mod 50),

3x≡12(mod 22)

?x ? 2 ?解:(1)化为?x ? 3 ??x ?1?mod 3?,?mod 5?,用孙子定理解,M=120,M1=40,M2=24,M3=15, ?mod 8?.令40 M1′≡1(mod 3), 得M1′≡1 (mod 3);

24 M2′≡1(mod 5), 得-M2′≡1(mod 5),M2′≡-1(mod 5); 15 M3′≡1(mod 8), 得-M2′≡1(mod 8),M2′≡-1(mod 8)

所以同余方程组的解是x≡40×2 + 24× (-1)×3 +15×(-1)×1 ≡ -7(mod 120).

?x ? 2 ?(3)化为?x ? 9 ??x ?3?mod 35?, ?,?mod1?mod 4?.??x ? 2 即??x ?3??mod 35?,

?mod 4?.用孙子定理解,M=140,M1=4,M2=35, 令4 M1′≡1(mod 35), 得M1′≡9 (mod 35);

35 M2′≡1(mod 4), 得-M2′≡1(mod 4),M2′≡-1(mod 4);

所以同余方程组的解是x≡4×9×2 + 35× (-1)×3 ≡ -33 ≡ 107 (mod 140).

6. 解我国古代数学家杨辉在 1275 年所写的《续古摘奇算法》中的三个例题: (1)七数剩一,八数剩一,九数剩三,问本数;

(2)十一数余三,十二数余二,十三数余一,问本数; (3)二数余一,五数余二,七数余三,九数余四,问本数.

??x ? 1 解:(1)???x ?3?mod 56?,

?mod 9?.方法一:设x=1+56y, 代入第二个同余方程,得

56y ≡ 3-1 (mod 13),(56-54)y ≡ 2 (mod 13)

得y ≡ 1 (mod 13) y,所以,同余方程组的解是x≡57 (mod 504)。 方法二:用孙子定理解。

?x ? 3 ?(2)?x ? 2 ??x ?1 ?,?mod11 ?,用孙子定理解,M=1716,M1=156,M2=143,M3=132, ?mod12 ?.?mod13令156 M1′≡1(mod 11), 得2M1′≡12(mod 11),M1′≡6(mod 11);

143 M2′≡1(mod 12), 得-M2′≡1(mod 12),M2′≡-1(mod 12); 132 M3′≡1(mod 13), 得2M2′≡-12(mod 13),M2′≡-6(mod 13)

所以同余方程组的解是x≡156×6×3 + 143× (-1)×2 +132×(-6)×1 ≡ 14(mod 17). 法二:观察法。或累加试除法。

7. 设韩信所辖某部士兵共 26641人,在一次战斗中损失近百人. 休整时清查:1~3报数余 1,1~5报数余 3,1~ 7报数余 4. 问损失了多少人?

?x ? 1 ?mod 3?,??x ? 3 ?mod 5?,解:设还有士兵x人,由题设,得同余方程组?

?x ?4?mod 7?,?26541?x ?26600.?

由口诀,x≡70 + 21×3 + 15×4≡193+105×251≡ 26548 (mod 105) 26641-26548=93人。损失了93人。

?26641?x ? 1 ??26641?x ? 3 设损失了x人,得同余方程组??26641?x ?4?0?x ?100.??x ? 0 ?mod 3?,?mod 3?,?mod 5,???x ? 3 ?mod 5?,化为?

?mod 7?,?x ?2?mod 7?,?0?x ?100.?由口诀,x=21×3 + 15×2=93。损失了93人。

8. 求 7的倍数,使它分别被 2,3,4,5,6除时,余数都是 1.

解:设所求为7x, [2,3,4,5,6] =60,由题设,得7x ? 1 ?mod 60?,

用大衍求一术得x≡43 (mod 60), 7x=7×43=301, 故,所求为301+420t , t 为整数。 9. 求三个连续的自然数,使它们从小到大依次被 15,17,19 整除(写出其中最小的一组).

??15x?1 ? 0 解:由题设,得同余方程组???15x ?2?0 ?, ?,??mod17?x ? 9 ?mod17化为标准形式为? ?. ?.?mod19??x ?10?mod19用孙子定理解,M=321,M1=19,M2=17,

令19 M1′≡1(mod 17), 得M1′≡9(mod 17), 17 M2′≡1(mod 19), 得M2′≡9(mod 19),

所以同余方程组的解是x≡19×(-8)×9 + 17×9×10 ≡ 162 (mod 321). 本题所求的三个连续的自然数是162×15,162×15+1,162×15+2,即2430,2431,2432。